Partitioning the Reals into two Locally Uncountable, Dense SetsLocally non-enumerable dense subsets of...

A Paper Record is What I Hamper

Does Gita support doctrine of eternal samsara?

Which big number is bigger?

Relationship between strut and baselineskip

Why did C use the -> operator instead of reusing the . operator?

What term is being referred to with "reflected-sound-of-underground-spirits"?

a sore throat vs a strep throat vs strep throat

If a planet has 3 moons, is it possible to have triple Full/New Moons at once?

Is the claim "Employers won't employ people with no 'social media presence'" realistic?

How can Republicans who favour free markets, consistently express anger when they don't like the outcome of that choice?

How do I check if a string is entirely made of the same substring?

Pulling the rope with one hand is as heavy as with two hands?

555 timer FM transmitter

Get consecutive integer number ranges from list of int

Should the Death Curse affect an undead PC in the Tomb of Annihilation adventure?

Can I criticise the more senior developers around me for not writing clean code?

What is the philosophical significance of speech acts/implicature?

Critique of timeline aesthetic

How did Captain America manage to do this?

Pre-plastic human skin alternative

Partitioning the Reals into two Locally Uncountable, Dense Sets

Providing evidence of Consent of Parents for Marriage by minor in England in early 1800s?

Do I have an "anti-research" personality?

"Whatever a Russian does, they end up making the Kalashnikov gun"? Are there any similar proverbs in English?



Partitioning the Reals into two Locally Uncountable, Dense Sets


Locally non-enumerable dense subsets of RUncountable dense subset whose complement is also uncountable and denseA question about uncountable, dense sets in RA Question regarding disjoint dense setsThere is no uncountable collection of pairwise disjoint open sets in $mathbb R$Prove that an uncountable space with the countable-closed topology satisfies the countable chain condition.Is $mathbb{R}$ the disjoint union of finitely many congruent dense sets?Can a countable dense subset be split into two disjoint dense subsets?3 dense uncountable pairwise disjoint subsets of real lineIs the intersection of a dense set and a linear subspace in $mathbb R^n$ dense?













2












$begingroup$


Is it possible to find two disjoint subsets $X$ and $Y$ of $mathbb{R}$ such that both are dense in $mathbb{R}$ and both are locally uncountable?



By a locally uncountable set $X subset mathbb{R}$, I mean a set which has the property that if I take any nonempty open subset $U$ of $mathbb{R}$, then $U cap X$ has cardinality strictly larger than the natural numbers.










share|cite|improve this question









$endgroup$

















    2












    $begingroup$


    Is it possible to find two disjoint subsets $X$ and $Y$ of $mathbb{R}$ such that both are dense in $mathbb{R}$ and both are locally uncountable?



    By a locally uncountable set $X subset mathbb{R}$, I mean a set which has the property that if I take any nonempty open subset $U$ of $mathbb{R}$, then $U cap X$ has cardinality strictly larger than the natural numbers.










    share|cite|improve this question









    $endgroup$















      2












      2








      2





      $begingroup$


      Is it possible to find two disjoint subsets $X$ and $Y$ of $mathbb{R}$ such that both are dense in $mathbb{R}$ and both are locally uncountable?



      By a locally uncountable set $X subset mathbb{R}$, I mean a set which has the property that if I take any nonempty open subset $U$ of $mathbb{R}$, then $U cap X$ has cardinality strictly larger than the natural numbers.










      share|cite|improve this question









      $endgroup$




      Is it possible to find two disjoint subsets $X$ and $Y$ of $mathbb{R}$ such that both are dense in $mathbb{R}$ and both are locally uncountable?



      By a locally uncountable set $X subset mathbb{R}$, I mean a set which has the property that if I take any nonempty open subset $U$ of $mathbb{R}$, then $U cap X$ has cardinality strictly larger than the natural numbers.







      general-topology measure-theory examples-counterexamples






      share|cite|improve this question













      share|cite|improve this question











      share|cite|improve this question




      share|cite|improve this question










      asked 4 hours ago









      Charles HudginsCharles Hudgins

      3006




      3006






















          2 Answers
          2






          active

          oldest

          votes


















          3












          $begingroup$

          Yes. For instance, for each interval with rational endpoints, pick a Cantor set in the interval, and let $X$ be the union of all these Cantor sets. Then $X$ has uncountable intersection with every open interval, but it is meager (or measure zero, if the Cantor sets you chose all have measure zero) so its complement $Y$ has uncountable intersection with every open interval as well.



          Here's another construction which gives uncountably many such subsets at once. For each $rinmathbb{R}$, let $X_r$ be the set of real numbers whose decimal expansion agrees with $r$'s decimal expansion on all but finitely many of the even digits. Then each $X_r$ has uncountably many points in any open interval, but $X_r$ and $X_s$ are disjoint unless all but finitely many of the even digits of $r$ agree with those of $s$ (in which case $X_r=X_s$).



          With heavy use of the axiom of choice, you can get even crazier examples. For instance, you can construct a family of uncountably many disjoint subsets of $mathbb{R}$, each of which has uncountable intersection with every uncountable closed subset of $mathbb{R}$. As a sketch of the proof, note that there are $2^{aleph_0}$ such uncountable closed sets and each has $2^{aleph_0}$ elements, so you can one by one pick elements to put in each of the sets you're building in a transfinite recursion of length $2^{aleph_0}$. For more details of this and related constructions, look up "Bernstein sets".






          share|cite|improve this answer











          $endgroup$





















            2












            $begingroup$

            Here is a choice free version. Let $A$ be the set of reals whose decimal expansions eventually consist of only $3$s and $4$s and $B$ be the set of reals whose decimal expansions eventually consist of only $5$s and $6$s. These are both locally uncountable dense subsets of $Bbb R$ and there are lots of reals I haven't used.






            share|cite|improve this answer









            $endgroup$














              Your Answer








              StackExchange.ready(function() {
              var channelOptions = {
              tags: "".split(" "),
              id: "69"
              };
              initTagRenderer("".split(" "), "".split(" "), channelOptions);

              StackExchange.using("externalEditor", function() {
              // Have to fire editor after snippets, if snippets enabled
              if (StackExchange.settings.snippets.snippetsEnabled) {
              StackExchange.using("snippets", function() {
              createEditor();
              });
              }
              else {
              createEditor();
              }
              });

              function createEditor() {
              StackExchange.prepareEditor({
              heartbeatType: 'answer',
              autoActivateHeartbeat: false,
              convertImagesToLinks: true,
              noModals: true,
              showLowRepImageUploadWarning: true,
              reputationToPostImages: 10,
              bindNavPrevention: true,
              postfix: "",
              imageUploader: {
              brandingHtml: "Powered by u003ca class="icon-imgur-white" href="https://imgur.com/"u003eu003c/au003e",
              contentPolicyHtml: "User contributions licensed under u003ca href="https://creativecommons.org/licenses/by-sa/3.0/"u003ecc by-sa 3.0 with attribution requiredu003c/au003e u003ca href="https://stackoverflow.com/legal/content-policy"u003e(content policy)u003c/au003e",
              allowUrls: true
              },
              noCode: true, onDemand: true,
              discardSelector: ".discard-answer"
              ,immediatelyShowMarkdownHelp:true
              });


              }
              });














              draft saved

              draft discarded


















              StackExchange.ready(
              function () {
              StackExchange.openid.initPostLogin('.new-post-login', 'https%3a%2f%2fmath.stackexchange.com%2fquestions%2f3203999%2fpartitioning-the-reals-into-two-locally-uncountable-dense-sets%23new-answer', 'question_page');
              }
              );

              Post as a guest















              Required, but never shown

























              2 Answers
              2






              active

              oldest

              votes








              2 Answers
              2






              active

              oldest

              votes









              active

              oldest

              votes






              active

              oldest

              votes









              3












              $begingroup$

              Yes. For instance, for each interval with rational endpoints, pick a Cantor set in the interval, and let $X$ be the union of all these Cantor sets. Then $X$ has uncountable intersection with every open interval, but it is meager (or measure zero, if the Cantor sets you chose all have measure zero) so its complement $Y$ has uncountable intersection with every open interval as well.



              Here's another construction which gives uncountably many such subsets at once. For each $rinmathbb{R}$, let $X_r$ be the set of real numbers whose decimal expansion agrees with $r$'s decimal expansion on all but finitely many of the even digits. Then each $X_r$ has uncountably many points in any open interval, but $X_r$ and $X_s$ are disjoint unless all but finitely many of the even digits of $r$ agree with those of $s$ (in which case $X_r=X_s$).



              With heavy use of the axiom of choice, you can get even crazier examples. For instance, you can construct a family of uncountably many disjoint subsets of $mathbb{R}$, each of which has uncountable intersection with every uncountable closed subset of $mathbb{R}$. As a sketch of the proof, note that there are $2^{aleph_0}$ such uncountable closed sets and each has $2^{aleph_0}$ elements, so you can one by one pick elements to put in each of the sets you're building in a transfinite recursion of length $2^{aleph_0}$. For more details of this and related constructions, look up "Bernstein sets".






              share|cite|improve this answer











              $endgroup$


















                3












                $begingroup$

                Yes. For instance, for each interval with rational endpoints, pick a Cantor set in the interval, and let $X$ be the union of all these Cantor sets. Then $X$ has uncountable intersection with every open interval, but it is meager (or measure zero, if the Cantor sets you chose all have measure zero) so its complement $Y$ has uncountable intersection with every open interval as well.



                Here's another construction which gives uncountably many such subsets at once. For each $rinmathbb{R}$, let $X_r$ be the set of real numbers whose decimal expansion agrees with $r$'s decimal expansion on all but finitely many of the even digits. Then each $X_r$ has uncountably many points in any open interval, but $X_r$ and $X_s$ are disjoint unless all but finitely many of the even digits of $r$ agree with those of $s$ (in which case $X_r=X_s$).



                With heavy use of the axiom of choice, you can get even crazier examples. For instance, you can construct a family of uncountably many disjoint subsets of $mathbb{R}$, each of which has uncountable intersection with every uncountable closed subset of $mathbb{R}$. As a sketch of the proof, note that there are $2^{aleph_0}$ such uncountable closed sets and each has $2^{aleph_0}$ elements, so you can one by one pick elements to put in each of the sets you're building in a transfinite recursion of length $2^{aleph_0}$. For more details of this and related constructions, look up "Bernstein sets".






                share|cite|improve this answer











                $endgroup$
















                  3












                  3








                  3





                  $begingroup$

                  Yes. For instance, for each interval with rational endpoints, pick a Cantor set in the interval, and let $X$ be the union of all these Cantor sets. Then $X$ has uncountable intersection with every open interval, but it is meager (or measure zero, if the Cantor sets you chose all have measure zero) so its complement $Y$ has uncountable intersection with every open interval as well.



                  Here's another construction which gives uncountably many such subsets at once. For each $rinmathbb{R}$, let $X_r$ be the set of real numbers whose decimal expansion agrees with $r$'s decimal expansion on all but finitely many of the even digits. Then each $X_r$ has uncountably many points in any open interval, but $X_r$ and $X_s$ are disjoint unless all but finitely many of the even digits of $r$ agree with those of $s$ (in which case $X_r=X_s$).



                  With heavy use of the axiom of choice, you can get even crazier examples. For instance, you can construct a family of uncountably many disjoint subsets of $mathbb{R}$, each of which has uncountable intersection with every uncountable closed subset of $mathbb{R}$. As a sketch of the proof, note that there are $2^{aleph_0}$ such uncountable closed sets and each has $2^{aleph_0}$ elements, so you can one by one pick elements to put in each of the sets you're building in a transfinite recursion of length $2^{aleph_0}$. For more details of this and related constructions, look up "Bernstein sets".






                  share|cite|improve this answer











                  $endgroup$



                  Yes. For instance, for each interval with rational endpoints, pick a Cantor set in the interval, and let $X$ be the union of all these Cantor sets. Then $X$ has uncountable intersection with every open interval, but it is meager (or measure zero, if the Cantor sets you chose all have measure zero) so its complement $Y$ has uncountable intersection with every open interval as well.



                  Here's another construction which gives uncountably many such subsets at once. For each $rinmathbb{R}$, let $X_r$ be the set of real numbers whose decimal expansion agrees with $r$'s decimal expansion on all but finitely many of the even digits. Then each $X_r$ has uncountably many points in any open interval, but $X_r$ and $X_s$ are disjoint unless all but finitely many of the even digits of $r$ agree with those of $s$ (in which case $X_r=X_s$).



                  With heavy use of the axiom of choice, you can get even crazier examples. For instance, you can construct a family of uncountably many disjoint subsets of $mathbb{R}$, each of which has uncountable intersection with every uncountable closed subset of $mathbb{R}$. As a sketch of the proof, note that there are $2^{aleph_0}$ such uncountable closed sets and each has $2^{aleph_0}$ elements, so you can one by one pick elements to put in each of the sets you're building in a transfinite recursion of length $2^{aleph_0}$. For more details of this and related constructions, look up "Bernstein sets".







                  share|cite|improve this answer














                  share|cite|improve this answer



                  share|cite|improve this answer








                  edited 3 hours ago

























                  answered 3 hours ago









                  Eric WofseyEric Wofsey

                  194k14223354




                  194k14223354























                      2












                      $begingroup$

                      Here is a choice free version. Let $A$ be the set of reals whose decimal expansions eventually consist of only $3$s and $4$s and $B$ be the set of reals whose decimal expansions eventually consist of only $5$s and $6$s. These are both locally uncountable dense subsets of $Bbb R$ and there are lots of reals I haven't used.






                      share|cite|improve this answer









                      $endgroup$


















                        2












                        $begingroup$

                        Here is a choice free version. Let $A$ be the set of reals whose decimal expansions eventually consist of only $3$s and $4$s and $B$ be the set of reals whose decimal expansions eventually consist of only $5$s and $6$s. These are both locally uncountable dense subsets of $Bbb R$ and there are lots of reals I haven't used.






                        share|cite|improve this answer









                        $endgroup$
















                          2












                          2








                          2





                          $begingroup$

                          Here is a choice free version. Let $A$ be the set of reals whose decimal expansions eventually consist of only $3$s and $4$s and $B$ be the set of reals whose decimal expansions eventually consist of only $5$s and $6$s. These are both locally uncountable dense subsets of $Bbb R$ and there are lots of reals I haven't used.






                          share|cite|improve this answer









                          $endgroup$



                          Here is a choice free version. Let $A$ be the set of reals whose decimal expansions eventually consist of only $3$s and $4$s and $B$ be the set of reals whose decimal expansions eventually consist of only $5$s and $6$s. These are both locally uncountable dense subsets of $Bbb R$ and there are lots of reals I haven't used.







                          share|cite|improve this answer












                          share|cite|improve this answer



                          share|cite|improve this answer










                          answered 3 hours ago









                          Ross MillikanRoss Millikan

                          302k24201375




                          302k24201375






























                              draft saved

                              draft discarded




















































                              Thanks for contributing an answer to Mathematics Stack Exchange!


                              • Please be sure to answer the question. Provide details and share your research!

                              But avoid



                              • Asking for help, clarification, or responding to other answers.

                              • Making statements based on opinion; back them up with references or personal experience.


                              Use MathJax to format equations. MathJax reference.


                              To learn more, see our tips on writing great answers.




                              draft saved


                              draft discarded














                              StackExchange.ready(
                              function () {
                              StackExchange.openid.initPostLogin('.new-post-login', 'https%3a%2f%2fmath.stackexchange.com%2fquestions%2f3203999%2fpartitioning-the-reals-into-two-locally-uncountable-dense-sets%23new-answer', 'question_page');
                              }
                              );

                              Post as a guest















                              Required, but never shown





















































                              Required, but never shown














                              Required, but never shown












                              Required, but never shown







                              Required, but never shown

































                              Required, but never shown














                              Required, but never shown












                              Required, but never shown







                              Required, but never shown







                              Popular posts from this blog

                              Paper upload error, “Upload failed: The top margin is 0.715 in on page 3, which is below the required...

                              Emraan Hashmi Filmografia | Linki zewnętrzne | Menu nawigacyjneGulshan GroverGulshan...

                              How can I write this formula?newline and italics added with leqWhy does widehat behave differently if I...